(8 intermediate revisions by the same user not shown)
Line 4: Line 4:
 
[[Category:problem solving]]
 
[[Category:problem solving]]
 
[[Category:image processing]]
 
[[Category:image processing]]
= [[ECE_PhD_Qualifying_Exams|ECE Ph.D. Qualifying Exam]] in Communication Networks Signal and Image processing (CS) =
 
= [[QE637_sol2013|Question 5, August 2013(Published on May 2017)]], Problem 1 =
 
 
:[[QE637_sol2013_Q1| Solution 1]],[[QE637_sol2013_Q1|2]]
 
  
 
<center>
 
<center>
Line 22: Line 18:
 
August 2013 (Published on May 2017)
 
August 2013 (Published on May 2017)
 
</center>
 
</center>
----
 
Question is posted from this [https://engineering.purdue.edu/ECE/Academics/Graduates/Archived_QE_August_13/CS-5.pdf <u>link</u>].<br>
 
----
 
 
 
== Problem 1  ==
 
== Problem 1  ==
  
Line 46: Line 38:
 
<math>P_{0}(e^{j\omega}) = \sum_{n=-\infty}^{\infty} p_{0}(n)e^{-jn\omega}</math><br>  
 
<math>P_{0}(e^{j\omega}) = \sum_{n=-\infty}^{\infty} p_{0}(n)e^{-jn\omega}</math><br>  
  
<math>P_{1}(e^{j\omega}) = \sum_{m=-\infty}^{\infty} p_{0}(m)e^{-jm\omega}</math><br>\\
+
<math>P_{1}(e^{j\omega}) = \sum_{m=-\infty}^{\infty} p_{1}(m)e^{-jm\omega}</math><br>
 
</center>
 
</center>
  
Line 57: Line 49:
 
d) Do the function&nbsp;<span class="texhtml">''p''<sub>0</sub>(''n'')</span>&nbsp;and&nbsp;<span class="texhtml">''p''<sub>1</sub>(''m'')</span>&nbsp;together contains sufficient information to reconstruction the function&nbsp;<span class="texhtml">''x''(''m'',''n'')</span>? If so, provide a reconstruction algorithm; if not, provide a counter example.  
 
d) Do the function&nbsp;<span class="texhtml">''p''<sub>0</sub>(''n'')</span>&nbsp;and&nbsp;<span class="texhtml">''p''<sub>1</sub>(''m'')</span>&nbsp;together contains sufficient information to reconstruction the function&nbsp;<span class="texhtml">''x''(''m'',''n'')</span>? If so, provide a reconstruction algorithm; if not, provide a counter example.  
  
Click [[QE637 2013 Pro1|here]] to view student [[QE637 2013 Pro1|answers and discussions]] <br>  
+
Click [[QE637_sol2013_Q1 |here]] to view student [[QE637_sol2013_Q1 |answers and discussions]] <br>  
  
 
----
 
----
  
<br> '''Problem 2. ''' (50 pts)
+
<br> '''Problem 2. '''
  
Let <span class="texhtml">''r''<sub>0</sub>(λ)</span>, <span class="texhtml">''g''<sub>0</sub>(λ)</span>, and <span class="texhtml">''b''<sub>0</sub>(λ)</span> be the CIE color matching functions for red, green, and blue primaries at 700 nm, 546.1 nm, and 435.8 nm, respectively, and let <span class="texhtml">[''r'',''g'',''b'']</span>&nbsp;be the corresponding CIE tristimulus values.&nbsp;&lt;/span&gt;
+
Let <span class="texhtml">''r''<sub>0</sub>(λ)</span>, <span class="texhtml">''g''<sub>0</sub>(λ)</span>, and <span class="texhtml">''b''<sub>0</sub>(λ)</span> be the CIE color matching functions for red, green, and blue primaries at 700 nm, 546.1 nm, and 435.8 nm, respectively, and let <span class="texhtml">[''r'',''g'',''b'']</span>&nbsp;be the corresponding CIE tristimulus values.
  
 
Furthermore, let&nbsp;<span class="texhtml">''f''<sub>1</sub>(λ)</span>,&nbsp;<span class="texhtml">''f''<sub>2</sub>(λ)</span>, and&nbsp;<span class="texhtml">''f''<sub>3</sub>(λ)</span>&nbsp;be the spectral response functions for the three color outputs of a color camera. So for each pixel of the camera sensor, there is a 3-dimensional output vector given by&nbsp;<span class="texhtml">''F'' = [''F''<sub>1</sub>,''F''<sub>2</sub>,''F''<sub>3</sub>]<sup>''t''</sup></span>, where  
 
Furthermore, let&nbsp;<span class="texhtml">''f''<sub>1</sub>(λ)</span>,&nbsp;<span class="texhtml">''f''<sub>2</sub>(λ)</span>, and&nbsp;<span class="texhtml">''f''<sub>3</sub>(λ)</span>&nbsp;be the spectral response functions for the three color outputs of a color camera. So for each pixel of the camera sensor, there is a 3-dimensional output vector given by&nbsp;<span class="texhtml">''F'' = [''F''<sub>1</sub>,''F''<sub>2</sub>,''F''<sub>3</sub>]<sup>''t''</sup></span>, where  
 
+
<center>
 
<math>F_1 = \int_{-\infty}^{\infty}f_1(\lambda)I(\lambda)d\lambda</math>,  
 
<math>F_1 = \int_{-\infty}^{\infty}f_1(\lambda)I(\lambda)d\lambda</math>,  
  
Line 72: Line 64:
  
 
<math>F_3 = \int_{-\infty}^{\infty}f_3(\lambda)I(\lambda)d\lambda</math>  
 
<math>F_3 = \int_{-\infty}^{\infty}f_3(\lambda)I(\lambda)d\lambda</math>  
 +
</center>
  
where&nbsp;<span class="texhtml">''I''(λ)</span>&nbsp;is the energy spectrum of the incoming light and&nbsp;<math>f_k(\lambda)\geq 0</math>&nbsp;for&nbsp;<span class="texhtml">''k'' = 0,1,2.</span>.
+
where&nbsp;<span class="texhtml">''I''(λ)</span>&nbsp;is the energy spectrum of the incoming light and&nbsp;<math>f_k(\lambda)\geq 0</math>&nbsp;for&nbsp;<span class="texhtml">''k'' = 0,1,2.</span>
  
 
Furthermore, assume there exists a matrix,&nbsp;<span class="texhtml">''M''</span>, so that  
 
Furthermore, assume there exists a matrix,&nbsp;<span class="texhtml">''M''</span>, so that  
 
+
<center>
 
<math>
 
<math>
 
\left[ {\begin{array}{*{20}{c}}
 
\left[ {\begin{array}{*{20}{c}}
Line 90: Line 83:
 
\end{array}} \right]
 
\end{array}} \right]
 
</math>  
 
</math>  
 +
</center>
  
 
<br> a) Why is it necessary that&nbsp;<math>f_k(\lambda) \geq 0</math>&nbsp;for&nbsp;<span class="texhtml">''k'' = 0,1,2</span>?  
 
<br> a) Why is it necessary that&nbsp;<math>f_k(\lambda) \geq 0</math>&nbsp;for&nbsp;<span class="texhtml">''k'' = 0,1,2</span>?  
Line 99: Line 93:
 
d) Do functions <span class="texhtml">''f''<sub>''k''</sub>(λ)</span> exist, which meet these requirements? If so, give a specific example of such functions.  
 
d) Do functions <span class="texhtml">''f''<sub>''k''</sub>(λ)</span> exist, which meet these requirements? If so, give a specific example of such functions.  
  
Click [[QE637 2013 Pro2|here]] to view student [[QE637 2013 Pro2|answers and discussions]]  
+
Click [[QE637_sol2013_Q2|here]] to view student [[QE637_sol2013_Q2|answers and discussions]]  
  
 
<br>  
 
<br>  
  
 
[[Category:ECE]] [[Category:QE]] [[Category:CNSIP]] [[Category:Problem_solving]] [[Category:Image_processing]]
 
[[Category:ECE]] [[Category:QE]] [[Category:CNSIP]] [[Category:Problem_solving]] [[Category:Image_processing]]

Latest revision as of 20:27, 2 May 2017


ECE Ph.D. Qualifying Exam

Communication, Networking, Signal and Image Processing (CS)

Question 5: Image Processing

August 2013 (Published on May 2017)

Problem 1

Consider the 2D discrete space signal x(m,n) with the DSFT of X(ejμ,ejν) given by 

$ X(e^{j\mu},e^{j\nu}) = \sum_{m=-\infty}^{\infty} \sum_{n=-\infty}^{\infty} x(m,n)e^{-j(m\mu+n\nu)} $

Then define

$ p_{0}(n) = \sum_{m=-\infty}^{\infty}x(m,n) $

$ p_{1}(m) = \sum_{n=-\infty}^{\infty}x(m,n) $

with corresponding DTFT given by 

$ P_{0}(e^{j\omega}) = \sum_{n=-\infty}^{\infty} p_{0}(n)e^{-jn\omega} $

$ P_{1}(e^{j\omega}) = \sum_{m=-\infty}^{\infty} p_{1}(m)e^{-jm\omega} $

a) Derive an expression for P0(ejω) in terms of X(ejμ,wjν). 

b) Derive an expression P0(ejω) in terms of X(ejμ,ejν).

c) Derive an expression  for $ \sum_{n = -\infty}^{\infty}p_0(n) $ interms of X(ejμ,ejν).

d) Do the function p0(n) and p1(m) together contains sufficient information to reconstruction the function x(m,n)? If so, provide a reconstruction algorithm; if not, provide a counter example.

Click here to view student answers and discussions



Problem 2.

Let r0(λ), g0(λ), and b0(λ) be the CIE color matching functions for red, green, and blue primaries at 700 nm, 546.1 nm, and 435.8 nm, respectively, and let [r,g,b] be the corresponding CIE tristimulus values.

Furthermore, let f1(λ)f2(λ), and f3(λ) be the spectral response functions for the three color outputs of a color camera. So for each pixel of the camera sensor, there is a 3-dimensional output vector given by F = [F1,F2,F3]t, where

$ F_1 = \int_{-\infty}^{\infty}f_1(\lambda)I(\lambda)d\lambda $,

$ F_2 = \int_{-\infty}^{\infty}f_2(\lambda)I(\lambda)d\lambda $,

$ F_3 = \int_{-\infty}^{\infty}f_3(\lambda)I(\lambda)d\lambda $

where I(λ) is the energy spectrum of the incoming light and $ f_k(\lambda)\geq 0 $ for k = 0,1,2.

Furthermore, assume there exists a matrix, M, so that

$ \left[ {\begin{array}{*{20}{c}} f_1(\lambda)\\ f_1(\lambda)\\ f_1(\lambda) \end{array}} \right] = {\begin{array}{*{20}{c}} M \end{array}} \left[ {\begin{array}{*{20}{c}} r_0(\lambda)\\ g_0(\lambda)\\ b_0(\lambda) \end{array}} \right] $


a) Why is it necessary that $ f_k(\lambda) \geq 0 $ for k = 0,1,2?

b) Are the functions, $ r_0(\lambda) \geq 0 $, $ g_0(\lambda) \geq 0 $, and $ b_0(\lambda) \geq 0 $? If so, why? If not, why not?

c) Derive an formula for the tristimulus vector [r,g,b]t in terms of the tristimulus vector F = [F1,F2,F3]t.

d) Do functions fk(λ) exist, which meet these requirements? If so, give a specific example of such functions.

Click here to view student answers and discussions


Alumni Liaison

Ph.D. 2007, working on developing cool imaging technologies for digital cameras, camera phones, and video surveillance cameras.

Buyue Zhang